Finding the limit of a function with absolute values.

Click For Summary
The limit of the function as x approaches 1 is analyzed using one-sided limits, yielding results of 3 from the right and 1 from the left, indicating the limit does not exist. A correction is made, confirming the left-hand limit is actually 1, but the overall conclusion remains that the limit does not exist due to differing one-sided limits. Further simplification of the function leads to a consistent result of -1 when evaluated correctly. However, there are concerns about the clarity of the calculations presented, particularly regarding notation and formatting. The discussion emphasizes the importance of proper limit evaluation and clear expression in mathematical communication.
drunkenfool
Messages
29
Reaction score
0
I need a little help and reassurance here.

The question is as follows,

Find the following limit, if it exists.

\lim_{x \rightarrow 1} \frac{x ^ 2 + |x -1| - 1}{|x-1|}

Here is what I did, first I did the two one-sided limits, as \lim_{x \rightarrow 1^+} and as \lim_{x \rightarrow 1^-}. (the values in the absolute value would be (x-1) and -(x-1) respectively, in this case) The answers I got were 3 and -1 respectively. Since the one-sided limits aren't the same, I concluded that the limit for this function does not exist. Am I correct?
 
Last edited:
Physics news on Phys.org
Makes sense to me.
 
One correction: the limit as x approaches 1 from below, \lim_{x \rightarrow 1^-}, is 1, not -1. Of course, the limit still doesn't exist.
 
Oh, how so? Am I doing this right?

=\frac{x ^ 2 + |x -1| - 1}{|x-1|}
=\frac{x ^ 2 + -(x -1) - 1}{-(x-1)}
=\frac{x ^ 2 -x}{-(x-1)}
=\frac{-x (1-x)}{1-x}
=-x
=-1
 
drunkenfool said:
Oh, how so? Am I doing this right?

=\frac{x ^ 2 + |x -1| - 1}{|x-1|}
=\frac{x ^ 2 + -(x -1) - 1}{-(x-1)}
=\frac{x ^ 2 -x}{-(x-1)}
=\frac{-x (1-x)}{1-x}
=-x
=-1
Yes, -1 is the correct answer. However, it's not very clear the way you wrote it. Why are your equal signs all fly off to the numerator, and where are all the lim notation?
----------------
Or you may try to get rid of the |x - 1| in the numerator first, and then apply the limit:
\lim_{x \rightarrow 1 ^ -} \frac{x ^ 2 - 1 + |x - 1|}{|x - 1|}
= 1 + \lim_{x \rightarrow 1 ^ -} \frac{x ^ 2 - 1}{|x - 1|}
= 1 - \lim_{x \rightarrow 1 ^ -} \frac{(x - 1)(x + 1)}{x - 1}
= 1 - \lim_{x \rightarrow 1 ^ -} (x + 1)
= -1.
 
Oh, I really don't know my way around the latex codes, so that's why you see all the errors. Thanks a lot, you guys.
 
Question: A clock's minute hand has length 4 and its hour hand has length 3. What is the distance between the tips at the moment when it is increasing most rapidly?(Putnam Exam Question) Answer: Making assumption that both the hands moves at constant angular velocities, the answer is ## \sqrt{7} .## But don't you think this assumption is somewhat doubtful and wrong?

Similar threads

  • · Replies 10 ·
Replies
10
Views
2K
  • · Replies 12 ·
Replies
12
Views
3K
  • · Replies 8 ·
Replies
8
Views
2K
  • · Replies 5 ·
Replies
5
Views
2K
  • · Replies 11 ·
Replies
11
Views
2K
  • · Replies 20 ·
Replies
20
Views
2K
Replies
5
Views
2K
  • · Replies 26 ·
Replies
26
Views
3K
  • · Replies 8 ·
Replies
8
Views
1K
  • · Replies 7 ·
Replies
7
Views
2K